How to prove that the process $Y_t$ is martingale?












4












$begingroup$


We have SDE $$dX_t = X_t(1-X_t)dW_t$$ where $W$ is standard Brownian motion and $X_0 = x_0 in (0,1)$. Assume that holds $P(X_t in (0,1))=1.$



For any $u$, we can define $f(x) = (frac{x}{1-x})^u sqrt{x(1-x)}$.



So first i have to prove that if we take $lambda = u^2 - frac{1}{4}$, for any $u$ the process $$Y_t = e^{frac{-lambda t}{2}}f(X_t)$$ is local martingale.



That part i did it. But now i have to take complex $u$. If i take $ui$ with real part $u$, then the process $Y_t$ is bounded and martingale. How i prove this?










share|cite|improve this question









$endgroup$












  • $begingroup$
    I don't understand what you mean by "If I take $ui$ with real part $u$ [...]"? Do you mean imaginary part? Do you want to consider $u=iv$ with $v$ real...?
    $endgroup$
    – saz
    Jan 30 at 18:59












  • $begingroup$
    Yes, the last part if i take $u = vi$ where $v$ is real number.
    $endgroup$
    – tjakra
    Jan 31 at 15:10
















4












$begingroup$


We have SDE $$dX_t = X_t(1-X_t)dW_t$$ where $W$ is standard Brownian motion and $X_0 = x_0 in (0,1)$. Assume that holds $P(X_t in (0,1))=1.$



For any $u$, we can define $f(x) = (frac{x}{1-x})^u sqrt{x(1-x)}$.



So first i have to prove that if we take $lambda = u^2 - frac{1}{4}$, for any $u$ the process $$Y_t = e^{frac{-lambda t}{2}}f(X_t)$$ is local martingale.



That part i did it. But now i have to take complex $u$. If i take $ui$ with real part $u$, then the process $Y_t$ is bounded and martingale. How i prove this?










share|cite|improve this question









$endgroup$












  • $begingroup$
    I don't understand what you mean by "If I take $ui$ with real part $u$ [...]"? Do you mean imaginary part? Do you want to consider $u=iv$ with $v$ real...?
    $endgroup$
    – saz
    Jan 30 at 18:59












  • $begingroup$
    Yes, the last part if i take $u = vi$ where $v$ is real number.
    $endgroup$
    – tjakra
    Jan 31 at 15:10














4












4








4





$begingroup$


We have SDE $$dX_t = X_t(1-X_t)dW_t$$ where $W$ is standard Brownian motion and $X_0 = x_0 in (0,1)$. Assume that holds $P(X_t in (0,1))=1.$



For any $u$, we can define $f(x) = (frac{x}{1-x})^u sqrt{x(1-x)}$.



So first i have to prove that if we take $lambda = u^2 - frac{1}{4}$, for any $u$ the process $$Y_t = e^{frac{-lambda t}{2}}f(X_t)$$ is local martingale.



That part i did it. But now i have to take complex $u$. If i take $ui$ with real part $u$, then the process $Y_t$ is bounded and martingale. How i prove this?










share|cite|improve this question









$endgroup$




We have SDE $$dX_t = X_t(1-X_t)dW_t$$ where $W$ is standard Brownian motion and $X_0 = x_0 in (0,1)$. Assume that holds $P(X_t in (0,1))=1.$



For any $u$, we can define $f(x) = (frac{x}{1-x})^u sqrt{x(1-x)}$.



So first i have to prove that if we take $lambda = u^2 - frac{1}{4}$, for any $u$ the process $$Y_t = e^{frac{-lambda t}{2}}f(X_t)$$ is local martingale.



That part i did it. But now i have to take complex $u$. If i take $ui$ with real part $u$, then the process $Y_t$ is bounded and martingale. How i prove this?







stochastic-processes stochastic-calculus martingales local-martingales






share|cite|improve this question













share|cite|improve this question











share|cite|improve this question




share|cite|improve this question










asked Jan 30 at 17:19









tjakratjakra

465




465












  • $begingroup$
    I don't understand what you mean by "If I take $ui$ with real part $u$ [...]"? Do you mean imaginary part? Do you want to consider $u=iv$ with $v$ real...?
    $endgroup$
    – saz
    Jan 30 at 18:59












  • $begingroup$
    Yes, the last part if i take $u = vi$ where $v$ is real number.
    $endgroup$
    – tjakra
    Jan 31 at 15:10


















  • $begingroup$
    I don't understand what you mean by "If I take $ui$ with real part $u$ [...]"? Do you mean imaginary part? Do you want to consider $u=iv$ with $v$ real...?
    $endgroup$
    – saz
    Jan 30 at 18:59












  • $begingroup$
    Yes, the last part if i take $u = vi$ where $v$ is real number.
    $endgroup$
    – tjakra
    Jan 31 at 15:10
















$begingroup$
I don't understand what you mean by "If I take $ui$ with real part $u$ [...]"? Do you mean imaginary part? Do you want to consider $u=iv$ with $v$ real...?
$endgroup$
– saz
Jan 30 at 18:59






$begingroup$
I don't understand what you mean by "If I take $ui$ with real part $u$ [...]"? Do you mean imaginary part? Do you want to consider $u=iv$ with $v$ real...?
$endgroup$
– saz
Jan 30 at 18:59














$begingroup$
Yes, the last part if i take $u = vi$ where $v$ is real number.
$endgroup$
– tjakra
Jan 31 at 15:10




$begingroup$
Yes, the last part if i take $u = vi$ where $v$ is real number.
$endgroup$
– tjakra
Jan 31 at 15:10










1 Answer
1






active

oldest

votes


















1












$begingroup$

If $y>0$ is a positive number and $u in mathbb{C}$ complex, then



$$y^u = exp(u log(y)).$$



Now if $u = i v$ for $v in mathbb{R}$, then this implies that



$$|y^u| = |exp(i v log(y))| leq 1 quad text{for all $y>0$} tag{1}$$



as $|e^{ix}|=1$ for any real $x$. Since the stochastic process satisfies, by assumption, $X_t in (0,1)$ almost surely, we can use $(1)$ for $y=X_t/(1-X_t)$ to obtain that $|f(X_t)| leq 1$ almost surely. In particular,



$$|Y_t| leq |f(X_t)| leq 1,$$



and this shows that $(Y_t)_{t geq 0}$ is bounded. Since any bounded local martingale is a "true" martingale you are done if you can show that $(Y_t)_{t geq 0}$ is a local martingale. To prove this, you can use exactly the same reasoning as you did in the real-valued case (i.e. apply Itô's formula and check that the process can be written as a stochastic integral with respect to Brownian motion).






share|cite|improve this answer









$endgroup$














    Your Answer





    StackExchange.ifUsing("editor", function () {
    return StackExchange.using("mathjaxEditing", function () {
    StackExchange.MarkdownEditor.creationCallbacks.add(function (editor, postfix) {
    StackExchange.mathjaxEditing.prepareWmdForMathJax(editor, postfix, [["$", "$"], ["\\(","\\)"]]);
    });
    });
    }, "mathjax-editing");

    StackExchange.ready(function() {
    var channelOptions = {
    tags: "".split(" "),
    id: "69"
    };
    initTagRenderer("".split(" "), "".split(" "), channelOptions);

    StackExchange.using("externalEditor", function() {
    // Have to fire editor after snippets, if snippets enabled
    if (StackExchange.settings.snippets.snippetsEnabled) {
    StackExchange.using("snippets", function() {
    createEditor();
    });
    }
    else {
    createEditor();
    }
    });

    function createEditor() {
    StackExchange.prepareEditor({
    heartbeatType: 'answer',
    autoActivateHeartbeat: false,
    convertImagesToLinks: true,
    noModals: true,
    showLowRepImageUploadWarning: true,
    reputationToPostImages: 10,
    bindNavPrevention: true,
    postfix: "",
    imageUploader: {
    brandingHtml: "Powered by u003ca class="icon-imgur-white" href="https://imgur.com/"u003eu003c/au003e",
    contentPolicyHtml: "User contributions licensed under u003ca href="https://creativecommons.org/licenses/by-sa/3.0/"u003ecc by-sa 3.0 with attribution requiredu003c/au003e u003ca href="https://stackoverflow.com/legal/content-policy"u003e(content policy)u003c/au003e",
    allowUrls: true
    },
    noCode: true, onDemand: true,
    discardSelector: ".discard-answer"
    ,immediatelyShowMarkdownHelp:true
    });


    }
    });














    draft saved

    draft discarded


















    StackExchange.ready(
    function () {
    StackExchange.openid.initPostLogin('.new-post-login', 'https%3a%2f%2fmath.stackexchange.com%2fquestions%2f3093820%2fhow-to-prove-that-the-process-y-t-is-martingale%23new-answer', 'question_page');
    }
    );

    Post as a guest















    Required, but never shown

























    1 Answer
    1






    active

    oldest

    votes








    1 Answer
    1






    active

    oldest

    votes









    active

    oldest

    votes






    active

    oldest

    votes









    1












    $begingroup$

    If $y>0$ is a positive number and $u in mathbb{C}$ complex, then



    $$y^u = exp(u log(y)).$$



    Now if $u = i v$ for $v in mathbb{R}$, then this implies that



    $$|y^u| = |exp(i v log(y))| leq 1 quad text{for all $y>0$} tag{1}$$



    as $|e^{ix}|=1$ for any real $x$. Since the stochastic process satisfies, by assumption, $X_t in (0,1)$ almost surely, we can use $(1)$ for $y=X_t/(1-X_t)$ to obtain that $|f(X_t)| leq 1$ almost surely. In particular,



    $$|Y_t| leq |f(X_t)| leq 1,$$



    and this shows that $(Y_t)_{t geq 0}$ is bounded. Since any bounded local martingale is a "true" martingale you are done if you can show that $(Y_t)_{t geq 0}$ is a local martingale. To prove this, you can use exactly the same reasoning as you did in the real-valued case (i.e. apply Itô's formula and check that the process can be written as a stochastic integral with respect to Brownian motion).






    share|cite|improve this answer









    $endgroup$


















      1












      $begingroup$

      If $y>0$ is a positive number and $u in mathbb{C}$ complex, then



      $$y^u = exp(u log(y)).$$



      Now if $u = i v$ for $v in mathbb{R}$, then this implies that



      $$|y^u| = |exp(i v log(y))| leq 1 quad text{for all $y>0$} tag{1}$$



      as $|e^{ix}|=1$ for any real $x$. Since the stochastic process satisfies, by assumption, $X_t in (0,1)$ almost surely, we can use $(1)$ for $y=X_t/(1-X_t)$ to obtain that $|f(X_t)| leq 1$ almost surely. In particular,



      $$|Y_t| leq |f(X_t)| leq 1,$$



      and this shows that $(Y_t)_{t geq 0}$ is bounded. Since any bounded local martingale is a "true" martingale you are done if you can show that $(Y_t)_{t geq 0}$ is a local martingale. To prove this, you can use exactly the same reasoning as you did in the real-valued case (i.e. apply Itô's formula and check that the process can be written as a stochastic integral with respect to Brownian motion).






      share|cite|improve this answer









      $endgroup$
















        1












        1








        1





        $begingroup$

        If $y>0$ is a positive number and $u in mathbb{C}$ complex, then



        $$y^u = exp(u log(y)).$$



        Now if $u = i v$ for $v in mathbb{R}$, then this implies that



        $$|y^u| = |exp(i v log(y))| leq 1 quad text{for all $y>0$} tag{1}$$



        as $|e^{ix}|=1$ for any real $x$. Since the stochastic process satisfies, by assumption, $X_t in (0,1)$ almost surely, we can use $(1)$ for $y=X_t/(1-X_t)$ to obtain that $|f(X_t)| leq 1$ almost surely. In particular,



        $$|Y_t| leq |f(X_t)| leq 1,$$



        and this shows that $(Y_t)_{t geq 0}$ is bounded. Since any bounded local martingale is a "true" martingale you are done if you can show that $(Y_t)_{t geq 0}$ is a local martingale. To prove this, you can use exactly the same reasoning as you did in the real-valued case (i.e. apply Itô's formula and check that the process can be written as a stochastic integral with respect to Brownian motion).






        share|cite|improve this answer









        $endgroup$



        If $y>0$ is a positive number and $u in mathbb{C}$ complex, then



        $$y^u = exp(u log(y)).$$



        Now if $u = i v$ for $v in mathbb{R}$, then this implies that



        $$|y^u| = |exp(i v log(y))| leq 1 quad text{for all $y>0$} tag{1}$$



        as $|e^{ix}|=1$ for any real $x$. Since the stochastic process satisfies, by assumption, $X_t in (0,1)$ almost surely, we can use $(1)$ for $y=X_t/(1-X_t)$ to obtain that $|f(X_t)| leq 1$ almost surely. In particular,



        $$|Y_t| leq |f(X_t)| leq 1,$$



        and this shows that $(Y_t)_{t geq 0}$ is bounded. Since any bounded local martingale is a "true" martingale you are done if you can show that $(Y_t)_{t geq 0}$ is a local martingale. To prove this, you can use exactly the same reasoning as you did in the real-valued case (i.e. apply Itô's formula and check that the process can be written as a stochastic integral with respect to Brownian motion).







        share|cite|improve this answer












        share|cite|improve this answer



        share|cite|improve this answer










        answered Jan 31 at 18:43









        sazsaz

        82.1k862131




        82.1k862131






























            draft saved

            draft discarded




















































            Thanks for contributing an answer to Mathematics Stack Exchange!


            • Please be sure to answer the question. Provide details and share your research!

            But avoid



            • Asking for help, clarification, or responding to other answers.

            • Making statements based on opinion; back them up with references or personal experience.


            Use MathJax to format equations. MathJax reference.


            To learn more, see our tips on writing great answers.




            draft saved


            draft discarded














            StackExchange.ready(
            function () {
            StackExchange.openid.initPostLogin('.new-post-login', 'https%3a%2f%2fmath.stackexchange.com%2fquestions%2f3093820%2fhow-to-prove-that-the-process-y-t-is-martingale%23new-answer', 'question_page');
            }
            );

            Post as a guest















            Required, but never shown





















































            Required, but never shown














            Required, but never shown












            Required, but never shown







            Required, but never shown

































            Required, but never shown














            Required, but never shown












            Required, but never shown







            Required, but never shown







            Popular posts from this blog

            Can a sorcerer learn a 5th-level spell early by creating spell slots using the Font of Magic feature?

            ts Property 'filter' does not exist on type '{}'

            mat-slide-toggle shouldn't change it's state when I click cancel in confirmation window